Last visit was: 19 Nov 2025, 06:11 It is currently 19 Nov 2025, 06:11
Close
GMAT Club Daily Prep
Thank you for using the timer - this advanced tool can estimate your performance and suggest more practice questions. We have subscribed you to Daily Prep Questions via email.

Customized
for You

we will pick new questions that match your level based on your Timer History

Track
Your Progress

every week, we’ll send you an estimated GMAT score based on your performance

Practice
Pays

we will pick new questions that match your level based on your Timer History
Not interested in getting valuable practice questions and articles delivered to your email? No problem, unsubscribe here.
Close
Request Expert Reply
Confirm Cancel
User avatar
TooLegitToQuit
Joined: 28 Mar 2005
Last visit: 04 Apr 2007
Posts: 32
Own Kudos:
Posts: 32
Kudos: 2
Kudos
Add Kudos
Bookmarks
Bookmark this Post
User avatar
tkirk32
Joined: 07 Mar 2005
Last visit: 01 Jul 2005
Posts: 32
Own Kudos:
Posts: 32
Kudos: 15
Kudos
Add Kudos
Bookmarks
Bookmark this Post
User avatar
sparky
Joined: 18 Apr 2005
Last visit: 30 Jul 2005
Posts: 321
Own Kudos:
Location: Canuckland
Posts: 321
Kudos: 102
Kudos
Add Kudos
Bookmarks
Bookmark this Post
User avatar
rthothad
Joined: 03 Nov 2004
Last visit: 14 Feb 2009
Posts: 315
Own Kudos:
Posts: 315
Kudos: 111
Kudos
Add Kudos
Bookmarks
Bookmark this Post
I will go with A
Statement 1: (2/3)^2x < 1 for any positive integer
Statement 2: Depending on x's value it could be either.
User avatar
TooLegitToQuit
Joined: 28 Mar 2005
Last visit: 04 Apr 2007
Posts: 32
Own Kudos:
Posts: 32
Kudos: 2
Kudos
Add Kudos
Bookmarks
Bookmark this Post
The OA is A.

I was confused because I read first statement as X = 2Y... doh~
User avatar
ywilfred
Joined: 07 Jul 2004
Last visit: 06 Mar 2012
Posts: 1,989
Own Kudos:
Location: Singapore
Posts: 1,989
Kudos: 2,031
Kudos
Add Kudos
Bookmarks
Bookmark this Post
A) If x = 1, y = 2, then 4(1/9) < 1
If x = 2, y = 4, then 16(1/81) < 1

A is sufficient

B) is not sufficient since no information about x is given.

Ans: A
User avatar
WinWinMBA
Joined: 20 Apr 2005
Last visit: 22 Oct 2005
Posts: 344
Own Kudos:
Posts: 344
Kudos: 2,614
Kudos
Add Kudos
Bookmarks
Bookmark this Post
tkirk32
A = insufficient

0 and positive numbers are fine. But the equation is greater then 1 if x and y are negative.
.


The question stem specifically states that both X and Y are positive integers. So you cannot use negative numbers as examples.

A is thus sufficient because you'll always get fractions with values < 1.

Test using Y= 1,2,3,4...etc to see it.
User avatar
gmat2me2
Joined: 18 Feb 2005
Last visit: 13 Jan 2006
Posts: 356
Own Kudos:
Posts: 356
Kudos: 21
Kudos
Add Kudos
Bookmarks
Bookmark this Post
sparky
TooLegitToQuit
IF X and Y are positive integers, is (4^X) (1/3)^Y < 1?

(A) Y= 2X

(B) Y = 4

A. 1)(4/9)^X < 1 if X >1 suff

2) is insuff since X can vary


Sparky that was good way to approach this problem.....

Yes it is A
User avatar
AJB77
Joined: 30 May 2005
Last visit: 23 Sep 2008
Posts: 236
Own Kudos:
Posts: 236
Kudos: 51
Kudos
Add Kudos
Bookmarks
Bookmark this Post
S1

4^X * (1/3)^2X = 2^2X / 3^2X = (2/3)^2X which is < 1 when X is a positive integer.

So Strike out B,C,E

S2

4^X * 1/3^4 = 4^X/81.This is < 1 if 1 <= X <= 3 and > 1 if X > 3

Strike out D

Answer is A
avatar
mandy
Joined: 30 May 2005
Last visit: 18 Sep 2005
Posts: 151
Own Kudos:
Posts: 151
Kudos: 216
Kudos
Add Kudos
Bookmarks
Bookmark this Post
:-D got C
st1 Y=2X does not give any value for x and y
insufficient


St2 y=4 DOES Not tell much except 1/3^4 = 1/81
that a may be answer depending on x
hence insufficient


TOgether yes we know Y= 4 AND Y/2 = x hence x=2
HENCE we can plug back the x and y values to answer the question
:wink:
avatar
mandy
Joined: 30 May 2005
Last visit: 18 Sep 2005
Posts: 151
Own Kudos:
Posts: 151
Kudos: 216
Kudos
Add Kudos
Bookmarks
Bookmark this Post
TooLegitToQuit
The OA is A.

I was confused because I read first statement as X = 2Y... doh~


it seems I am wrong can anyone explain me why the first statement is sufficient thanks :?
User avatar
gumbico
Joined: 15 Apr 2005
Last visit: 07 Jul 2005
Posts: 10
Own Kudos:
Location: Fairfax, VA
Posts: 10
Kudos: 1
Kudos
Add Kudos
Bookmarks
Bookmark this Post
ywilfred's explanation covers what I did but here's just detailed breakdown:

-from the stem, we know X and Y are positive so we only need to test pos numbers

(A) Y= 2X

-from this, we know Y is twice X. I used (x=1,y=2) and(x=2,y=4). You can select any number so long as it fits (A). (x=5,y=10) etc. But small numbers are easier to work with.

Quote:
A) If x = 1, y = 2, then 4(1/9) < 1
If x = 2, y = 4, then 16(1/81) < 1

A is sufficient


Now the only possible choices are A and D.

(B) Y=4

-this says nothing about X so it's insufficient

Answer is A.

The logic you are following (two unknowns require two equations) doesn't apply here because we're not asked to find a specific value for either X or Y. We only need to know if

(4^X) (1/3)^Y < 1

And from (A), we know that the left side of this inequality will always be less than 1.



Archived Topic
Hi there,
This topic has been closed and archived due to inactivity or violation of community quality standards. No more replies are possible here.
Where to now? Join ongoing discussions on thousands of quality questions in our Data Sufficiency (DS) Forum
Still interested in this question? Check out the "Best Topics" block above for a better discussion on this exact question, as well as several more related questions.
Thank you for understanding, and happy exploring!
Moderators:
Math Expert
105388 posts
GMAT Tutor
1924 posts